LSAT and Law School Admissions Forum

Get expert LSAT preparation and law school admissions advice from PowerScore Test Preparation.

 Administrator
PowerScore Staff
  • PowerScore Staff
  • Posts: 8917
  • Joined: Feb 02, 2011
|
#27119
Complete Question Explanation
(The complete setup for this game can be found here: lsat/viewtopic.php?t=11115)

The correct answer choice is (A)

The question stem specifies that H is not offered. As the fourth rule lists H as a necessary condition, taking the contrapositive of the fourth rule reveals that G cannot be offered. G is not one of the answer choices, however. But, G is a necessary condition of the second rule, and taking the contrapositive of that rule reveals that L cannot be offered:
June 06_M12_game#3_L6_explanations_game#4_#15_diagram_1.png

Accordingly, answer choice (A) is correct.
You do not have the required permissions to view the files attached to this post.
 lsatbossintraining
  • Posts: 27
  • Joined: Oct 21, 2019
|
#71684
Hm. Shouldn’t M be kicked out as well?

Or is that because L relates to M in an “or” relationship that M could actually be in while all of H, G and L are out.

Feel like I understand it intuitively but looking for someone to cement the idea for me.

Thanks much.

Kyle
 Jeremy Press
PowerScore Staff
  • PowerScore Staff
  • Posts: 1000
  • Joined: Jun 12, 2017
|
#71696
Hi Kyle,

Yes, you've got the right idea here! If M is in, then one of L or S must be in (though not both). Even if L is out, M could still be in, so long as S is also in.

Here's how we would diagram the rule about M, along with its contrapositive:
Screen Shot 2019-11-04 at 1.17.00 PM.png
We'd have to know that both L and S were out before we could say with certainty that M is out.

I hope this helps!

Jeremy
You do not have the required permissions to view the files attached to this post.

Get the most out of your LSAT Prep Plus subscription.

Analyze and track your performance with our Testing and Analytics Package.